Are Sigma-Finite Measures Always Lebesgue Measurable?

  • Thread starter Thread starter island-boy
  • Start date Start date
Click For Summary
The discussion revolves around two problems related to sigma-finite measures and Lebesgue measurability. In the first question, the user attempts to demonstrate that if a Lebesgue measurable function f is positive on a measure space (X, M, m), then the measure m is sigma-finite, concluding that since m(X) is finite, each subset Xi must also have finite measure. In the second question, the user seeks to construct a Lebesgue measurable function f that is positive and integrates to 1 over a sigma-finite measure space, expressing uncertainty about how to eliminate the m(X) term in their formulation. The user is looking for confirmation of their reasoning and assistance in refining their approach to the second problem. Overall, the discussion highlights the relationship between sigma-finite measures and Lebesgue measurable functions.
island-boy
Messages
93
Reaction score
0
hello, I need some help with 2 problesm involving sigma-finite measures.

*note* a set X is sigma-finite if X can be written as a compact union of subsets, i.e. X = Union of Xi for i = 1,2,3,...n, and the measure of each Xi is finite.

q1)Given a measure space (X, M, m), let f be lebesgue measurable such that f(x)>0 for every x element of X, show that m is sigma-finite.
----
I think I may have solved this. Please see if I did anything wrong.

Let X = union of arbitrary disjoint subsets Xi where i =1,2,3...n

since f is lebesgue measurable, then f(x)m(X) is finite, hence m(X) is finite.
Since m(X) = m (UXi) = summation m(Xi) is finite. Hence m(Xi) is finite.
hence m is sigma finite. Is this correct?

----------

q2) suppose f is sigma-finite, constrauct a lebesgue measurable function s.t. f(x) > 0 for every x element of X abd the integral of f(x)dx over X = 1.

-----
I'm thinking this is how to solve this.
since the integral of f(x)dx over X = 1 then
f(x)m(X) = 1
and
f(x) = 1/m(X)...this f(x) is finite, hence it is lebesgue meaurable.

I'm thinking I need to get rid of the m(x) part, but I have no idea how. help please?
 
Physics news on Phys.org
anyone? help please. thanks!
 
Question: A clock's minute hand has length 4 and its hour hand has length 3. What is the distance between the tips at the moment when it is increasing most rapidly?(Putnam Exam Question) Answer: Making assumption that both the hands moves at constant angular velocities, the answer is ## \sqrt{7} .## But don't you think this assumption is somewhat doubtful and wrong?

Similar threads

Replies
1
Views
2K
Replies
15
Views
2K
  • · Replies 2 ·
Replies
2
Views
2K
  • · Replies 1 ·
Replies
1
Views
2K
  • · Replies 3 ·
Replies
3
Views
1K
  • · Replies 3 ·
Replies
3
Views
3K
Replies
34
Views
3K
  • · Replies 2 ·
Replies
2
Views
2K
  • · Replies 4 ·
Replies
4
Views
1K
Replies
6
Views
2K